Difference between revisions of "User talk:Bobthesmartypants/Sandbox"

(Picture 1)
(Picture 2)
Line 75: Line 75:
 
</asy>
 
</asy>
 
<cmath>\text{Prove the shaded areas are equal.}</cmath>
 
<cmath>\text{Prove the shaded areas are equal.}</cmath>
 +
==sndbozx==
 +
<asy>
 +
draw((9,0)--(21,0)--(16,12)--(0,12)--cycle);
 +
draw(Circle((12,6),6));
 +
draw((12,0)--(12,12),linetype("8 8")+red);
 +
draw((12,6)--(228/13,108/13),linetype("8 8")+red);
 +
draw((12,6)--(9-27/(sqrt(313)),36/(sqrt(313))),linetype("8 8")+red);
 +
label("$A$",(0,12),N);
 +
label("$B$",(16,12),N);
 +
label("$C$",(21,0),S);
 +
label("$D$",(9,0),S);
 +
label("$E$",(12,12),N,red);
 +
label("$F$",(228/13,108/13),NE,red);
 +
label("$G$",(12,0),S,red);
 +
label("$H$",(9-27/(sqrt(313)),36/(sqrt(313))),SW,red);
 +
label("$O$",(12,6),NW);
 +
draw((16,12)--(16,0),linetype("8 8")+green);
 +
label("$P$",(16,0),S,green);
 +
</asy>
 +
We are given that <math>AB=16</math>, <math>CD=12</math>, and <math>\overline{AB}\parallel\overline{CD}</math>.
 +
 +
We can forget the restriction <math>BC<AD</math> because if <math>BC>AD</math>, we can just switch the labeling around so that <math>BC<AD</math>.
 +
 +
Label the center of the inscribed circle <math>O</math>; Draw lines <math>\overline{OE}</math>, <math>\overline{OF}</math>, <math>\overline{OG}</math>, and <math>\overline{OH}</math>
 +
 +
where <math>E</math>, <math>F</math>, <math>G</math>, and <math>H</math> are the tangent points of the circle and <math>\overline{AB}</math>, <math>\overline{BC}</math>, <math>\overline{CD}</math> and <math>\overline{DA}</math> respectively.
 +
 +
Note that <math>\angle EBF+\angle GCF=180^{\circ}</math> because the angles of quadrilateral <math>EGCB</math> add up to <math>360^{\circ}</math>, and <math>\angle OEB=\angle OGC=90^{\circ}</math>.
 +
 +
Also, <math>\angle EBF+\angle EOF=180^{\circ}</math> because the angles of quadrilateral <math>EBFO</math> add up to <math>360^{\circ}</math>, and <math>\angle OEB=\angle OFB=90^{\circ}</math>.
 +
 +
Therefore, <math>\angle GCF=\angle EOF</math>. By a similar reasoning, <math>\angle EBF=\angle GOF</math>. Therefore, <math>EBFO\sim GOFC</math>.
 +
 +
By a similar reasoning as the right side, <math>AEOH\sim OGDH</math>.
 +
 +
Let <math>BE=a</math>, and <math>AE=b</math>, <math>DG=c</math>, and <math>CG=d</math>. We also know that <math>EO=FO=GO=HO=6</math> because the diameter of circle <math>O</math> is <math>12</math>.
 +
 +
Since <math>EBFO\sim GOFC</math>, then <math>\dfrac{c}{GO}=\dfrac{EO}{b}\implies\dfrac{c}{6}=\dfrac{6}{b}\implies bc=36</math>.
 +
 +
Similarly, since <math>AEOH\sim OGDH</math>, then <math>\dfrac{d}{GO}=\dfrac{EO}{a}\implies\dfrac{d}{6}=\dfrac{6}{a}\implies ad=36</math>.
 +
 +
Also, <math>a+b=AB=16</math>, and <math>c+d=CD=12</math>. Therefore <math>b=16-a</math> and <math>d=12-c</math>.
 +
 +
We now substitute <math>b</math> and <math>d</math> into our other two equations: <math>(16-a)c=36</math> and <math>a(12-c)=36</math>.
 +
 +
Expanding gives <math>16c-ac=36</math> and <math>12a-ac=36</math>. Subtracting these two equations gives <math>12a-16c=0\implies 12a=16c\implies a=\dfrac{4}{3}c</math>.
 +
 +
Substituting <math>a</math> back into <math>a(12-c)=36</math> yields <math>\dfrac{4}{3}c(12-c)=36\implies 16c-\dfrac{4}{3}c^2=36\implies \dfrac{4}{3}c^2-16c+36=0</math>
 +
 +
Solving this quadratic gives <math>c=3, 9</math>. Based the the picture, <math>c</math> is obviously not <math>9</math> since <math>c<d</math>, so <math>c=3</math>. Therefore, <math>d=12-c=9</math>.
 +
 +
(Note: if I had said <math>c=9</math>, there wouldn't be any contradiction, apart from the fact that the picture would be drawn "flipped", i.e not to scale.)
 +
 +
Also, <math>a=\dfrac{4}{3}(3)=4</math> and so <math>b=16-a=12</math>.
 +
 +
All we need to find now is the length of <math>\overline{BC}</math>. Draw the height <math>\overline{BP}</math> with base <math>\overline{CD}</math>.
 +
 +
Since <math>\angle BPC=90^{\circ}</math>, we can use Pythagorean Theorem: <math>PC=d-a=5</math>, <math>BP=12</math>, therefore <math>BC=\sqrt{5^2+12^2}=\boxed{13}</math>. <math>\Box</math>

Revision as of 23:08, 21 October 2013

Bobthesmartypants's Sandbox

Solution 1

[asy] path Q; Q=(0,0)--(1,2)--(5,2)--(4,0)--cycle; draw(Q); draw((0,0)--(1.5,1)); label("D",(0,0),S); draw((1,2)--(1.5,1)); label("A",(1,2),N); draw((5,2)--(1.5,1)); label("B",(5,2),N); draw((4,0)--(1.5,1)); label("C",(4,0),S); draw((2,0)--(1.5,1),linetype("8 8")); label("E",(2,0),S); draw((2/3,4/3)--(1.5,1),linetype("8 8")); label("F",(2/3,4/3),W); label("P",(1.5,1),NNE); [/asy]

First, continue $\overline{AP}$ to hit $\overline{CD}$ at $E$. Also continue $\overline{CP}$ to hit $\overline{AD}$ at $F$.

We have that $\angle PAB=\angle PCB$. Because $\overline{AB}\parallel\overline{CD}$, we have $\angle PAB=\angle PED$.

Similarly, because $\overline{AD}\parallel\overline{BC}$, we have $\angle PCB=\angle PFD$.

Therefore, $\angle PAB=\angle PED=\angle PCB=\angle PFD$.

We also have that $\angle ADC=\angle ABC$ because $ABCD$ is a parallelogram, and $\angle APC=\angle FPE$.

Therefore, $ABCP\sim FDEP$. This means that $\dfrac{FD}{AB}=\dfrac{FP}{AP}=\dfrac{DP}{BP}$, so $\Delta ABP\sim\Delta FDP$.

Therefore, $\angle PBA=\angle PDA$. $\Box$


Solution 2

Note that $\dfrac{1}{n}$ is rational and $n$ is not divisible by $2$ nor $5$ because $n>11$.

This means the decimal representation of $\dfrac{1}{n}$ is a repeating decimal.

Let us set $a_1a_2\cdots a_x$ as the block that repeats in the repeating decimal: $\dfrac{1}{n}=0.\overline{a_1a_2\cdots a_x}$.

($a_1a_2\cdots a_x$ written without the overline used to signify one number so won't confuse with notation for repeating decimal)

The fractional representation of this repeating decimal would be $\dfrac{1}{n}=\dfrac{a_1a_2\cdots a_x}{10^x-1}$.

Taking the reciprocal of both sides you get $n=\dfrac{10^x-1}{a_1a_2\cdots a_x}$.

Multiplying both sides by $a_1a_2\cdots a_n$ gives $n(a_1a_2\cdots a_x)=10^x-1$.

Since $10^x-1=9\times \underbrace{111\cdots 111}_{x\text{ times}}$ we divide $9$ on both sides of the equation to get $\dfrac{n(a_1a_2\cdots a_x)}{9}=\underbrace{111\cdots 111}_{x\text{ times}}$.

Because $n$ is not divisible by $3$ (therefore $9$) since $n>11$ and $n$ is prime, it follows that $n|\underbrace{111\cdots 111}_{x\text{ times}}$. $\Box$

Picture 1

[asy]draw(Circle((1,1),2)); draw(Circle((sqrt(2),sqrt(3)/2),1)); dot((8/5,2/5)); dot((1,1)); draw((1,1)--(8/5,2/5),linetype("8 8")); label("a",(6/5,7/10),SSW); draw((8/5,2/5)--(12/5,-2/5),linetype("8 8")); label("b",(2,0),SSW); [/asy] \[\text{Find the probability that }b>a \text{.}\]

Picture 2

[asy] for (int i=0;i<6;i=i+1){ draw(dir(60*i)--dir(60*i+60)); } draw(dir(120)--(dir(0)+dir(-60))/2); draw(dir(180)--(dir(60)+dir(0))/2); fill(dir(120)--dir(180)--intersectionpoint(dir(120)--(dir(0)+dir(-60))/2,dir(180)--(dir(60)+dir(0))/2)--cycle,grey); fill((dir(0)+dir(-60))/2--dir(0)--(dir(60)+dir(0))/2--intersectionpoint(dir(120)--(dir(0)+dir(-60))/2,dir(180)--(dir(60)+dir(0))/2)--cycle,grey); [/asy] \[\text{Prove the shaded areas are equal.}\]

sndbozx

[asy] draw((9,0)--(21,0)--(16,12)--(0,12)--cycle); draw(Circle((12,6),6)); draw((12,0)--(12,12),linetype("8 8")+red); draw((12,6)--(228/13,108/13),linetype("8 8")+red); draw((12,6)--(9-27/(sqrt(313)),36/(sqrt(313))),linetype("8 8")+red); label("$A$",(0,12),N); label("$B$",(16,12),N); label("$C$",(21,0),S); label("$D$",(9,0),S); label("$E$",(12,12),N,red); label("$F$",(228/13,108/13),NE,red); label("$G$",(12,0),S,red); label("$H$",(9-27/(sqrt(313)),36/(sqrt(313))),SW,red); label("$O$",(12,6),NW); draw((16,12)--(16,0),linetype("8 8")+green); label("$P$",(16,0),S,green); [/asy] We are given that $AB=16$, $CD=12$, and $\overline{AB}\parallel\overline{CD}$.

We can forget the restriction $BC<AD$ because if $BC>AD$, we can just switch the labeling around so that $BC<AD$.

Label the center of the inscribed circle $O$; Draw lines $\overline{OE}$, $\overline{OF}$, $\overline{OG}$, and $\overline{OH}$

where $E$, $F$, $G$, and $H$ are the tangent points of the circle and $\overline{AB}$, $\overline{BC}$, $\overline{CD}$ and $\overline{DA}$ respectively.

Note that $\angle EBF+\angle GCF=180^{\circ}$ because the angles of quadrilateral $EGCB$ add up to $360^{\circ}$, and $\angle OEB=\angle OGC=90^{\circ}$.

Also, $\angle EBF+\angle EOF=180^{\circ}$ because the angles of quadrilateral $EBFO$ add up to $360^{\circ}$, and $\angle OEB=\angle OFB=90^{\circ}$.

Therefore, $\angle GCF=\angle EOF$. By a similar reasoning, $\angle EBF=\angle GOF$. Therefore, $EBFO\sim GOFC$.

By a similar reasoning as the right side, $AEOH\sim OGDH$.

Let $BE=a$, and $AE=b$, $DG=c$, and $CG=d$. We also know that $EO=FO=GO=HO=6$ because the diameter of circle $O$ is $12$.

Since $EBFO\sim GOFC$, then $\dfrac{c}{GO}=\dfrac{EO}{b}\implies\dfrac{c}{6}=\dfrac{6}{b}\implies bc=36$.

Similarly, since $AEOH\sim OGDH$, then $\dfrac{d}{GO}=\dfrac{EO}{a}\implies\dfrac{d}{6}=\dfrac{6}{a}\implies ad=36$.

Also, $a+b=AB=16$, and $c+d=CD=12$. Therefore $b=16-a$ and $d=12-c$.

We now substitute $b$ and $d$ into our other two equations: $(16-a)c=36$ and $a(12-c)=36$.

Expanding gives $16c-ac=36$ and $12a-ac=36$. Subtracting these two equations gives $12a-16c=0\implies 12a=16c\implies a=\dfrac{4}{3}c$.

Substituting $a$ back into $a(12-c)=36$ yields $\dfrac{4}{3}c(12-c)=36\implies 16c-\dfrac{4}{3}c^2=36\implies \dfrac{4}{3}c^2-16c+36=0$

Solving this quadratic gives $c=3, 9$. Based the the picture, $c$ is obviously not $9$ since $c<d$, so $c=3$. Therefore, $d=12-c=9$.

(Note: if I had said $c=9$, there wouldn't be any contradiction, apart from the fact that the picture would be drawn "flipped", i.e not to scale.)

Also, $a=\dfrac{4}{3}(3)=4$ and so $b=16-a=12$.

All we need to find now is the length of $\overline{BC}$. Draw the height $\overline{BP}$ with base $\overline{CD}$.

Since $\angle BPC=90^{\circ}$, we can use Pythagorean Theorem: $PC=d-a=5$, $BP=12$, therefore $BC=\sqrt{5^2+12^2}=\boxed{13}$. $\Box$